Which ratio represents the tangent of an angle?

a. adjacent/hypotenuse
b. opposite/hypotenuse
c. adjacent/opposite
d. opposite/adjacent

Answers

Answer 1

Answer:

option d.opposite / adjacent

Step-by-step explanation:

opposite /adjacent ratio represents the tangent of an angle .

hope it is helpful to you ☺️

Answer 2

Answer:

D.

Step-by-step explanation:

From the trigonometry shortcuts we can use the acronyms:

SOH  CAH  TOA

for an arbitrary angle Ф, plug in the length of the sides:

sin(Ф) = opposite/hypotenuse

cos(Ф) = adjacent/hypotenuse

tan(Ф) = opposite/adjacent


Related Questions

Given f(x) = 3sqrt(2x-1).
6(2x-1)^2-3

What is lim f(x)?

Answers

Answer:

[tex]\displaystyle 51[/tex]

General Formulas and Concepts:

Algebra I

Terms/CoefficientsFactoringFunctionsFunction Notation

Algebra II

Piecewise functions

Calculus

Limits

Right-Side Limit:                                                                                             [tex]\displaystyle \lim_{x \to c^+} f(x)[/tex]

Limit Rule [Variable Direct Substitution]:                                                             [tex]\displaystyle \lim_{x \to c} x = c[/tex]

Limit Property [Addition/Subtraction]:                                                                   [tex]\displaystyle \lim_{x \to c} [f(x) \pm g(x)] = \lim_{x \to c} f(x) \pm \lim_{x \to c} g(x)[/tex]

Limit Property [Multiplied Constant]:                                                                     [tex]\displaystyle \lim_{x \to c} bf(x) = b \lim_{x \to c} f(x)[/tex]

Step-by-step explanation:

Step 1: Define

Identify

[tex]\displaystyle f(x) = \left \{ {{3\sqrt{2x - 1}, \ x \leq 2} \atop {6(2x - 1)^2 - 3, \ x > 2}} \right.[/tex]

Step 2: Solve

Substitute in function [Limit]:                                                                         [tex]\displaystyle \lim_{x \to 2^+} 6(2x - 1)^2 - 3[/tex]Factor:                                                                                                           [tex]\displaystyle \lim_{x \to 2^+} 3[2(2x - 1)^2 - 1][/tex]Rewrite [Limit Property - Multiplied Constant]:                                           [tex]\displaystyle 3\lim_{x \to 2^+} 2(2x - 1)^2 - 1[/tex]Evaluate [Limit Property - Variable Direct Substitution]:                             [tex]\displaystyle 3[2(2 \cdot 2 - 1)^2 - 1][/tex]Simplify:                                                                                                         [tex]\displaystyle 51[/tex]

Topic: AP Calculus AB/BC (Calculus I/I + II)

Unit: Limits

Book: College Calculus 10e

The radius of a sphere is increasing at a rate of 3 mm/s. How fast is the volume increasing when the diameter is 60 mm

Answers

Answer:

The volume is increasing at a rate of 33929.3 cubic millimeters per second.

Step-by-step explanation:

Volume of a sphere:

The volume of a sphere of radius r is given by:

[tex]V = \frac{4\pi r^3}{3}[/tex]

In this question:

We have to derivate V and r implicitly in function of time, so:

[tex]\frac{dV}{dt} = 4\pi r^2\frac{dr}{dt}[/tex]

The radius of a sphere is increasing at a rate of 3 mm/s.

This means that [tex]\frac{dr}{dt} = 3[/tex]

How fast is the volume increasing when the diameter is 60 mm?

Radius is half the diameter, so [tex]r = 30[/tex]. We have to find [tex]\frac{dV}{dt}[/tex]. So

[tex]\frac{dV}{dt} = 4\pi r^2\frac{dr}{dt}[/tex]

[tex]\frac{dV}{dt} = 4\pi (30)^2(3) = 33929.3[/tex]

The volume is increasing at a rate of 33929.3 cubic millimeters per second.

Evaluate −3w − 6p for w=2 and p = −7

Answers

-3w-6p when w=2 and p=-7

-3(2)-6(-7)

= -6 + 42

= 36

Answer:

48

Step-by-step explanation:

-3w-6p when w=2 and p--7

you want to plug in the numbers to their letters

-3(2)-6(-7)

then you want to times the numbers.

-6-42

=48

Please help due tomorrow

Answers

Answer:

10x8=80 that would be the area for the picture 14x11=154 for the boards area

PLEASE HELP please I need this done now


The total cost of a truck rental, y, for x days, can be modeled by y = 35x + 25.
What is the rate of change for this function?

Answers
A- 35$
B-25$
C-60$
D-10$

Answers

Answer:

35

Step-by-step explanation:

y = 35x+23 is in the form

y = mx+b  where m is the slope and b is the y intercept

The slope can also be called the rate of change

35 is the slope

The answer to the question is A which is 35$

A cable that weighs 6 lb/ft is used to lift 600 lb of coal up a mine shaft 500 ft deep. Find the work done. Show how to approximate the required work by a Riemann sum. (Let x be the distance in feet below the top of the shaft. Enter xi* as xi.)

Answers

Answer:

A cable that weighs 6 lb/ft is used to lift 600 lb of coal up a mine shaft 500 ft deep. Find the work done. Show how to approximate the required work by a Riemann sum.

Step-by-step explanation:

Translate the triangle. Then enter the new coordinates. A(-3, 4) A'([?], [?]) B'([ ], [ ] C([],[]) B(0, 1) C(-4,1)

or

Answers

Answer:

The new coordinates are [tex]A'(x,y) = (3, 0)[/tex], [tex]B'(x,y) = (6, -3)[/tex] and [tex]C'(x,y) = (2, -3)[/tex].

Step-by-step explanation:

Vectorially speaking, the translation of a point can be defined by the following expression:

[tex]V'(x,y) = V(x,y) + T(x,y)[/tex] (1)

Where:

[tex]V(x,y)[/tex] - Original point.

[tex]V'(x,y)[/tex] - Translated point.

[tex]T(x,y)[/tex] - Translation vector.

If we know that [tex]A(x,y) = (-3,4)[/tex], [tex]B(x,y) = (0,1)[/tex], [tex]C(x,y) = (-4,1)[/tex] and [tex]T(x,y) = (6, -4)[/tex], then the resulting points are:

[tex]A'(x,y) = (-3, 4) + (6, -4)[/tex]

[tex]A'(x,y) = (3, 0)[/tex]

[tex]B'(x,y) = (0,1) + (6, -4)[/tex]

[tex]B'(x,y) = (6, -3)[/tex]

[tex]C'(x,y) = (-4, 1) + (6, -4)[/tex]

[tex]C'(x,y) = (2, -3)[/tex]

The new coordinates are [tex]A'(x,y) = (3, 0)[/tex], [tex]B'(x,y) = (6, -3)[/tex] and [tex]C'(x,y) = (2, -3)[/tex].

A street light is mounted at the top of a 15-ft-tall pole. A man 6 feet tall walks away from the pole with a speed of 5 ft/s along a straight path. How fast (in ft/s) is the tip of his shadow moving when he is 45 feet from the pole

Answers

Answer:

25/3 ft/s

Step-by-step explanation:

Height of pole , h=15 ft

Height of man, h'=6 ft

Let BD=x

BE=y

DE=BE-BD=y-x

All right triangles are similar

When two triangles are similar then the ratio of their corresponding sides are equal.

Therefore,

[tex]\frac{AB}{CD}=\frac{BE}{DE}[/tex]

[tex]\frac{15}{6}=\frac{y}{y-x}[/tex]

[tex]\frac{5}{2}=\frac{y}{y-x}[/tex]

[tex]5y-5x=2y[/tex]

[tex]5y-2y=5x[/tex]

[tex]3y=5x[/tex]

[tex]y=\frac{5}{3}x[/tex]

Differentiate w.r.t t

[tex]\frac{dy}{dt}=\frac{5}{3}\frac{dx}{dt}[/tex]

We have dx/dt=5ft/s

Using the value

[tex]\frac{dy}{dt}=\frac{5}{3}(5)=\frac{25}{3}ft/s[/tex]

Hence, the tip of  his shadow moving  with a speed 25/3 ft/s when he is 45 feet from the pole.

Answer:

The tip pf the shadow is moving with speed 25/3 ft/s.

Step-by-step explanation:

height of pole = 15 ft

height of man = 6 ft

x = 45 ft

According to the diagram, dx/dt = 5 ft/s.

Now

[tex]\frac{y-x}{y}=\frac{6}{15}\\\\15 y - 15 x = 6 y \\\\y = \frac{5}{3} x\\\\\frac{dy}{dt} = \frac{5}{3}\frac{dx}{dt}\\\\\frac{dy}{dt}=\frac{5}{3}\times 5 =\frac{25}{3} ft/s[/tex]

The combined value of the ages of Mary, Kate and Tom is 26 years. What will be their age in total after 2 years?

Answers

Answer:

32

Step-by-step explanation:

they will each age two years, 3x2 is 6, add 6 to 26

Answer:

32

Step-by-step explanation:

they will each age two years, 3x2 is 6, add 6 to 26

Simplify the expression. 8x^-10 y^'6 -2x^2y^-8 Write your answer without negative exponents.​

Answers

Answer:

[tex]8x^{-10}y^6 - 2x^2y^{-8} = \frac{8y^{14} - 2x^{12}}{x^{10}y^8}[/tex]

Step-by-step explanation:

Given

[tex]8x^{-10}y^6 - 2x^2y^{-8}[/tex]

Required

Simplify

Rewrite as:

[tex]8x^{-10}y^6 - 2x^2y^{-8} = \frac{8y^6}{x^{10}} - \frac{2x^2}{y^8}[/tex]

Take LCM

[tex]8x^{-10}y^6 - 2x^2y^{-8} = \frac{8y^6*y^8 - 2x^2 * x^{10}}{x^{10}y^8}[/tex]

Apply law of indices

[tex]8x^{-10}y^6 - 2x^2y^{-8} = \frac{8y^{14} - 2x^{12}}{x^{10}y^8}[/tex]

Given: F = {(0, 1), (2, 4), (4, 6), (6, 8)} and G = {(2, 5), (4, 7), (5, 8), (6, 9), (7, 5)}

(F + G) (2) =

4
5
9

Answers

9514 1404 393

Answer:

  9

Step-by-step explanation:

The ordered pair (2, 4) in the relation for function F tells you F(2) = 4.

The ordered pair (2, 5) in the relation for function G tells you G(2) = 5.

Then the sum is ...

  (F+G)(2) = F(2) +G(2) = 4 +5

  (F+G)(2) = 9

HELP ME WITH THIS MATHS QUESTION
PICTURE IS ATTACHED

Answers

Answer:

In picture.

Step-by-step explanation:

To do this answer, you need to count the boxes up to the mirror line. This will give us the exact place to draw the triangle.

The picture below is the answer.

Identify the domain of the function shown in the graph.

A. -5 B. x> 0
C. 0 D. x is all real numbers.

Answers

I believe it could be D. X is al, real numbers but correct me if I’m wrong please.

Determine la razón de la siguiente progresión geométrica: 81,27,9,3,1,....

Answers

Answer:

BẠN BỊ ĐIÊN À

Step-by-step explanation:

CÚT

according to the fundemental theorem of algebra, how many roots exist for the polynomial function? f(x) = (x^3-3x+1)^2

Answers

Answer:

6

Step-by-step explanation:

First, we can expand the function to get its expanded form and to figure out what degree it is. For a polynomial function with one variable, the degree is the largest exponent value (once fully expanded/simplified) of the entire function that is connected to a variable. For example, x²+1 has a degree of 2, as 2 is the largest exponent value connected to a variable. Similarly, x³+2^5 has a degree of 2 as 5 is not an exponent value connected to a variable.

Expanding, we get

(x³-3x+1)²  = (x³-3x+1)(x³-3x+1)

= x^6 - 3x^4 +x³ - 3x^4 +9x²-3x + x³-3x+1

= x^6 - 6x^4 + 2x³ +9x²-6x + 1

In this function, the largest exponential value connected to the variable, x, is 6. Therefore, this is to the 6th degree. The fundamental theorem of algebra states that a polynomial of degree n has n roots, and as this is of degree 6, this has 6 roots

Which of the following statements are correct? Select ALL that apply!
Select one or more:
O a. -1.430 = -1.43
O b. 2.36 < 2.362
O c.-1.142 < -1.241
O d.-2.33 > -2.29
O e. 2.575 < 2.59
O f. -2.25 -2.46

Answers

I believe the answer is d.

Log6^(4x-5)=Log6^(2x+1)

Answers

Answer:

[tex]x = 3[/tex]

Step-by-step explanation:

Given

[tex]\log6^{(4x-5)} =\log6^{(2x+1)}[/tex]

Required

Solve for x

We have:

[tex]\log6^{(4x-5)} =\log6^{(2x+1)}[/tex]

Remove log6 from both sides

[tex](4x-5) = (2x+1)[/tex]

Collect like terms

[tex]4x - 2x = 5 + 1[/tex]

[tex]2x = 6[/tex]

Divide by 2

[tex]x = 3[/tex]

What is the percent increase from 250 to 900?

1. Write the percent change formula for an increase.

Percent Increase =
Amount of Increase
Original Amount

2. Substitute the known quantities for the amount of the increase and the original amount.

Percent Increase =
900 − 250
250

3. Subtract.

Percent Increase =
650
250

Answers

Answer:

260% is the correct answer

Step-by-step explanation:

i hope i helped

X = The set of months in a year?

Answers

there are 12 set of months in a year

Two workers finished a job in 12 days. How long would it take each worker to do the job by himself if one of the workers needs 10 more days to finish the job than the other worker

Answers

Two workers finished a job in 7.5 days.

How long would it take each worker to do the job by himself if one of the workers needs 8 more days to finish the job than the other worker?

let t = time required by one worker to complete the job alone

then

(t+8) = time required by the other worker (shirker)

let the completed job = 1

A typical shared work equation

7.5%2Ft + 7.5%2F%28%28t%2B8%29%29 = 1

multiply by t(t+8), cancel the denominators, and you have

7.5(t+8) + 7.5t = t(t+8)

7.5t + 60 + 7.5t = t^2 + 8t

15t + 60 = t^2 + 8t

form a quadratic equation on the right

0 = t^2 + 8t - 15t - 60

t^2 - 7t - 60 = 0

Factor easily to

(t-12) (t+5) = 0

the positive solution is all we want here

t = 12 days, the first guy working alone

then

the shirker would struggle thru the job in 20 days.

Answer:7 + 17 = 24÷2 (since there are 2 workers) =12. Also, ½(7) + ½17 = 3.5 + 8.5 = 12. So, we know that the faster worker will take 7 days and the slower worker will take 17 days. Hope this helps! jul15

Step-by-step explanation:

Verify that the indicated family of functions is a solution of the given differential equation. Assume an appropriate interval I of definition for each solution.

d^2y/ dx^2 − 6 dy/dx + 9y = 0; y = c1e3x + c2xe3x When y = c1e3x + c2xe3x,

Answers

y'' - 6y' + 9y = 0

If y = C₁ exp(3x) + C₂ x exp(3x), then

y' = 3C₁ exp(3x) + C₂ (exp(3x) + 3x exp(3x))

y'' = 9C₁ exp(3x) + C₂ (6 exp(3x) + 9x exp(3x))

Substituting these into the DE gives

(9C₁ exp(3x) + C₂ (6 exp(3x) + 9x exp(3x)))

… … … - 6 (3C₁ exp(3x) + C₂ (exp(3x) + 3x exp(3x)))

… … … + 9 (C₁ exp(3x) + C₂ x exp(3x))

= 9C₁ exp(3x) + 6C₂ exp(3x) + 9C₂ x exp(3x))

… … … - 18C₁ exp(3x) - 6C₂ (exp(3x) - 18x exp(3x))

… … … + 9C₁ exp(3x) + 9C₂ x exp(3x)

= 0

so the provided solution does satisfy the DE.

Please help!!! Find the domain of the function y = 2 cot(5∕8x).
A) All real numbers except odd integer multiples of 8π∕5
B) All real numbers except 0 and integer multiples of 8π∕5
C) All real numbers except 0 and integer multiples of 4π∕5
D) All real numbers except odd integer multiples of 4π∕5

Answers

Answer:

B) All real numbers except 0 and integer multiples of 8π∕5

Step-by-step explanation:

Cotangent function:

The cotangent function is given by:

[tex]y = \cot{ax} = \frac{\cos{ax}}{\sin{ax}}[/tex]

Domain:

All real values except those at which:

[tex]\sin{ax} = 0[/tex]

The sine is 0 for 0 and all integer multiples of [tex]\frac{1}{a}[/tex]

In this question:

[tex]a = \frac{5}{8}[/tex], so the values outside the domain are 0 and the integer multiples of [tex]\frac{8}{5}[/tex]. Then the correct answer is given by option b.

How many tens are in 6 hundreds

Answers

The answer is 60 just 60

Answer:

60

Step-by-step explanation:

10 x 6 = 60

Hope this helped! :)

What is the endpoint of a line segment if the midpoint M( – 3, 4) and the other endpoint is E(7, – 2)?
Answers
(– 13, 10)
(10, – 13)
(– 1, 2)
(2, – 1)

Answers

9514 1404 393

Answer:

  (-13, 10)

Step-by-step explanation:

If M is the midpoint of segment DE, then ...

  D = 2M -E

  D = 2(-3, 4) -(7, -2) = (2(-3)-7, 2(4)+2) = (-13, 10)

The other end point is (-13, 10).

please help, it’s urgent !!!

Answers

D

A

B

C

for more explanation please don't hesitate to just respond

Hi, help with question 18 please. thanks​

Answers

Answer:

See Below.

Step-by-step explanation:

We are given the equation:

[tex]\displaystyle y^2 = 1 + \sin x[/tex]

And we want to prove that:

[tex]\displaystyle 2y\frac{d^2y}{dx^2} + 2\left(\frac{dy}{dx}\right) ^2 + y^2 = 1[/tex]

Find the first derivative by taking the derivative of both sides with respect to x:

[tex]\displaystyle 2y \frac{dy}{dx} = \cos x[/tex]

Divide both sides by 2y:

[tex]\displaystyle \frac{dy}{dx} = \frac{\cos x}{2y}[/tex]

Find the second derivative using the quotient rule:

[tex]\displaystyle \begin{aligned} \frac{d^2y}{dx^2} &= \frac{(\cos x)'(2y) - (\cos x)(2y)'}{(2y)^2}\\ \\ &= \frac{-2y\sin x-2\cos x \dfrac{dy}{dx}}{4y^2} \\ \\ &= -\frac{y\sin x + \cos x\left(\dfrac{\cos x}{2y}\right)}{2y^2} \\ \\ &= -\frac{2y^2\sin x+\cos ^2 x}{4y^3}\end{aligned}[/tex]

Substitute:

[tex]\displaystyle 2y\left(-\frac{2y^2\sin x+\cos ^2 x}{4y^3}\right) + 2\left(\frac{\cos x}{2y}\right)^2 +y^2 = 1[/tex]

Simplify:

[tex]\displaystyle \frac{-2y^2\sin x-\cos ^2x}{2y^2} + \frac{\cos ^2 x}{2y^2} + y^2 = 1[/tex]

Combine fractions:

[tex]\displaystyle \frac{\left(-2y^2\sin x -\cos^2 x\right)+\left(\cos ^2 x\right)}{2y^2} + y^2 = 1[/tex]

Simplify:

[tex]\displaystyle \frac{-2y^2\sin x }{2y^2} + y^2 = 1[/tex]

Cancel:

[tex]\displaystyle -\sin x + y^2 = 1[/tex]

Substitute:

[tex]-\sin x + \left( 1 + \sin x\right) =1[/tex]

Simplify. Hence:

[tex]1\stackrel{\checkmark}{=}1[/tex]

Q.E.D.

A bank records deposits as positive numbers and withdrawals as negative numbers.
Mike withdrew $60 from his bank account 3 times.
what is the change in mikes account balance after all 3 withdrawals?

Answers

answer: -180

work: 0-60-60-60

a+b=60000
[tex]\frac{a}{b}=\frac{4}{1}[/tex]
a=?
b=?

Answers

Answer: a = 25.67

Step-by-step explanation:

use the figure to find y

Answers

Answer:

y = 3

Step-by-step explanation:

6sin(30) = 3

Select the correct answer.
Each statement describes a transformation of the graph of y=x. Which statement correctly describes the graph of y= x - 13?
OA. It is the graph of y= x translated 13 units to the right.
OB. It is the graph of y=xwhere the slope is decreased by 13.
It is the graph of y= x translated 13 units to the left.
OD. It is the graph of y= x translated 13 units up.
ОС.

Answers

minus sign ironically makes it go to the right

because the function crosses the y axis at -13

It is the graph of y = x translated 13 units down is the statement describes a transformation of the graph of y=x.

What is Graph?

Graph is a mathematical representation of a network and it describes the relationship between lines and points.

The equation y = x - 13 represents a transformation of the graph of y = x. To find the type of  transformation, we have to compare the two equations and look for changes.

In the equation y = x - 13, we subtract 13 from the value of x.

This means that the graph of y = x is shifted 13 units downwards,

since every point on the graph has 13 subtracted from its y-coordinate.

Hence,  It is the graph of y = x translated 13 units down is the statement describes a transformation of the graph of y=x.

To learn more on Graph click:

https://brainly.com/question/17267403

#SPJ7

Other Questions
calcium hydrogen sulphite cress cross 5 importance of doctor in points Johanna wants to start a blog about her experiences as an online adult learner. When deciding whether to make her blog anonymous or to use her given name, Johanna should consider what a potential __________ might think about the blog. who are called Guthiyaar and thakali Write a three-paragraph essay with the title "The Tension Between Traditional Culture andModernization in the Contemporary World." Be sure to address these questions: (10 points) 9x m 2 + 6x + 1 cho mnh hi cu ny Un avin puede volar con la velocidad de 400 km por hora en atmsfera tranquila si cuando se dirige hacia el este el viento viene del Sur con la velocidad de 40 km por hora cul es la direccin de su vuelo factor and solve the problem in the photo . pleaseeee helppppp i havent done algebra in 2 years If the observed genotypes in a given population do not fit those predicted by the Hardy-Weinberg equation, it could mean that.. Explain how the existence of ions relates to the number of electrons present in atoms of an element? (A.P.E.X) The diagram shows three points P, Q and R on horizontal ground.PQ = 50 m, PR = 100 m and angie PQR = 140.Calculate angle PRO. Find the value of x in each case:Please help meIt's an easy 40 points if you answer this A changes16. By accident, 6 burned out bulbs have been mixed in with 16 good ones, Ken is replacing old bulbs in his house. If he selects two bulbs at random from the box of 22, what is theprobability they both work? It is harder to get in shape than staying in shape What inspirations did you have for the piece? For example, were you inspired by something you've seen or experienced, or did the piece come from something you imagined? Which of the following is true of living things that have prokaryotic cells when compared to living things that have eukaryotic cells?They have many types of organelles.They are types of plants.O They are always larger than eukaryotic organisms.They are always unicellular. A lease provides that a retail tenant pays $900 minimum rent per month plus 3% of the gross sales in excess of $200,000 per year. If the tenant paid a total rent of $20,520 last year, what was the gross sales volume What theory explains why the Uranian system is on its side? Use the confidence level and sample data to find a confidence interval for estimating the population . Round your answer to the same number of decimal places as the sample mean.Test scores: n = 92, = 90.6, = 8.9; 99% confidenceOptions:A.) 88.2 < < 93.0B.) 88.4 < < 92.8C.) 89.1 < < 92.1D.) 88.8 < < 92.4 can someone help me pls